Angle Chasing Will Not Help You Here

Geometry Level 5

In a triangle A B C ABC A = 8 4 , \angle A =84^\circ, C = 7 8 . \angle C=78^\circ. Points D D and E E are taken on the sides A B AB and B C , BC, so that A C D = 4 8 , \angle ACD =48^\circ, C A E = 6 3 . \angle CAE =63^\circ. What is the measure (in degrees) of C D E \angle CDE ?


The answer is 81.

This section requires Javascript.
You are seeing this because something didn't load right. We suggest you, (a) try refreshing the page, (b) enabling javascript if it is disabled on your browser and, finally, (c) loading the non-javascript version of this page . We're sorry about the hassle.

8 solutions

Eloy Machado
Apr 9, 2014

By angle chasing we find E C D = 30 ° , A E C = 39 ° , E A D = 21 ° , C D A = A C D = 48 ° \angle ECD = 30°, \angle AEC = 39°, \angle EAD = 21°, \angle CDA = \angle ACD = 48° . So A C D \triangle ACD is isosceles.

Let M M be the midpoint of C D CD . So, A M AM is the perpendicular bissector.

WLOG, let A C = A D = 1 AC=AD=1 . So, C M = M D = cos 48 ° CM=MD=\cos { 48° } and C D = 2 cos 48 ° CD=2\cos { 48° } . Let D E A \angle DEA be α \alpha .

By sine rule in C D E \triangle CDE the get: sin ( 39 ° + α ) C D = sin 30 ° E D sin ( 39 ° + α ) 2 cos 48 ° = sin 30 ° E D \frac { \sin { (39°+\alpha ) } }{ CD } =\frac { \sin { 30° } }{ ED } \Rightarrow \frac { \sin { (39°+\alpha ) } }{ 2\cos { 48° } } =\frac { \sin { 30° } }{ ED } .

By sine rule in A D E \triangle ADE the get: sin ( α ) A D = sin 21 ° E D sin ( α ) 1 = sin 21 ° E D \frac { \sin { (\alpha ) } }{ AD } =\frac { \sin { 21° } }{ ED } \Rightarrow \frac { \sin { (\alpha ) } }{ 1 } =\frac { \sin { 21° } }{ ED } .

Doing the math with the 2 above equations, we get: sin ( 39 ° + α ) cos 48 ° = sin ( α ) sin 21 ° \frac { \sin { (39°+\alpha ) } }{ \cos { 48° } } =\frac { \sin { (\alpha ) } }{ \sin { 21° } }

Since cos 48 ° = sin 42 ° = 2 sin 21 ° cos 21 ° \cos { 48° } =\sin { 42° } =2\sin { 21° } \cos { 21° } we can rewrite the last equation as: sin ( 39 ° + α ) sin ( α ) = 2 cos 21 ° \frac { \sin { (39°+\alpha ) } }{ \sin { (\alpha ) } } =2\cos { 21° }

Since sin ( 39 ° + α ) = sin 39 ° cos α + sin α cos 39 ° \sin { (39°+\alpha ) } = \sin { 39° } \cos { \alpha } +\sin { \alpha } \cos { 39° } we will get in left side of equation:

sin 39 ° cos α + sin α cos 39 ° sin α = sin 39 ° cot α + cos 39 ° \frac { \sin { 39° } \cos { \alpha } +\sin { \alpha } \cos { 39° } }{ \sin { \alpha } } =\sin { 39° } \cot { \alpha } +\cos { 39° }

Now, we will need some creativity in right side: Write 21 ° 21° as ( 60 ° 39 ° ) (60° - 39°) :

2 cos 21 ° = 2 cos ( 60 ° 39 ° ) = 2 ( cos 60 ° cos 39 ° + sin 60 ° sin 39 ° ) = cos 39 ° + 3 sin 39 ° 2\cos { 21° } = 2\cos { (60° - 39°) } = 2 \left( \cos { 60° } \cos { 39° } +\sin { 60° } \sin { 39° } \right) =\cos { 39° } +\sqrt { 3 } \sin { 39° } .

Now, the equation is:

sin 39 ° cot α + cos 39 ° = cos 39 ° + 3 sin 39 ° \sin { 39° } \cot { \alpha } +\cos { 39° } = \cos { 39° } +\sqrt { 3 } \sin { 39° }

Then:

sin 39 ° cot α = 3 sin 39 ° \sin { 39° } \cot { \alpha } = \sqrt { 3 } \sin { 39° }

and finally: cot α = 3 \cot { \alpha } = \sqrt { 3 } . Therefore, α = 30 ° \alpha = 30° .

Returning to the C D E \triangle CDE and plugging in α = 30 ° \alpha = 30° , we have 39 ° + 30 ° + C D E + 30 ° = 180 ° C D E = 81 ° 39° + 30° + \angle CDE + 30° = 180° \Rightarrow \angle CDE = \boxed{81°} .

And we are done.

it seems wrong assumption

Tahir Javed - 7 years, 1 month ago

Log in to reply

it is correct, theoratically as well as practically

Adnan Jutt - 7 years ago

According to the giving of the question, we have a fact that;

At triangle "CmA" and its angles are:

"ACm" = 48, "CAm" = 63 so Angel "CmA" = 69

Actually we can't consider or assume; AM to be perpendicular to DC

Hossam EL-Deen Moustafa - 7 years, 1 month ago

Log in to reply

Please, read carefully my solution. M isn't the intersection of CD and AE.

M is the midpoint of CD. Midpoint! MC = MA.

Eloy Machado - 7 years, 1 month ago

Log in to reply

How can we assume MC=MA and the angles ( ACM & CAM ) aren't equal

Kareem Nour - 7 years, 1 month ago

Log in to reply

@Kareem Nour I didn't assume it. My M is midpoint of CD.

Eloy Machado - 7 years, 1 month ago

angle CAm can't be 63 degree

Saurabh Wadhwani - 7 years, 1 month ago

This is a right approach to the solution. Many get confused about his AD=1 assumption because they may not realize that we can assume AD as any value without changing the value of any of the angles. For example you can assume AD as a random value : x, then follow his approach replacing 1( his assumption for AD) with a. Then you will get to the same answer.

Hổ Nổi Loạn - 7 years, 1 month ago

Good solution.. No mistake in solution, i found it correct.. but i suspect that more than one solution is possible for this question, as by angle chase there is only one linear equation is available having two variables. So i expect more than one solution.. although its a great work, i appreciate.

du dev - 7 years, 1 month ago

ur solution is true and easy to be understood @ Eloy Machado

Haidar Islam Bhuiyan - 7 years, 1 month ago

i agree with Sid Leo. if we assume that CED=90 and AEC=39 then AED will be =51; and if AMC=69 then DME will also be =69; so CDE=(180-(51+69)) or CDE=60

Kailon Ablaza - 7 years, 1 month ago

Just find all the angles of every triangle and every quadrilateral. Triangles have a total of 180 degrees. Quadrilaterals have 360 degrees.

Marquis Cane Loria - 7 years, 1 month ago

CDE= 48

Aqeel Ahmad - 7 years, 1 month ago

Log in to reply

I too got the same ans

Praveen Upadrasta - 7 years ago

yes same to you

Muntakimul Towhid - 7 years ago

by angle chasing we get : m(ACD)=m(ADC)=48 , m(DCE)=30 SINCE in triangle ACD m(ACD)=m(ADC)=48 then triangle ACD is iscoceles triangle then AC=AD then AC and AD are two radius in circle A , AC and AD are two tangents sutend the same chord DC then m(ECD)=m(EDC)=30

Ranim Amine - 7 years, 2 months ago

Log in to reply

your right it is impossible for us to break that theory since we can't assume any principles that has no solid proof in that situation, i agree to your answer my answer is also 30, hence i am also looking forward to the considered correct answer 81 degrees both of it may be the correct conclusion but by just looking to the formal reasoning of the solution of 81 degrees we cannot assume it's principles unless it does not portrays any actual 100% proof in that situation so 30 is the correct answer rather that 81 degrees.

Sigmund Dela Cruz - 7 years, 1 month ago

ha ha ha

Tufail Khan - 7 years, 1 month ago

wo nice one

Jamshed Saleem - 7 years, 2 months ago

nice

Taha Ahmed - 7 years, 1 month ago

i think 81 is wrong answer bcoz line CD & AE makes angle of 111

Jambhale Vaibhav - 7 years ago

79 is the answer... :(

Vladimir Reyes - 7 years ago

superb

Abinesh G - 7 years, 1 month ago

Too good...

Sayyam Jain - 7 years, 1 month ago

it is not perpendicular

Tahir Javed - 7 years, 1 month ago

It's too good, Sir. I couldn't think the way you've done it. It's awesome. Math is at its best!

সালাম বাবু - 7 years, 1 month ago

but sir how can u assume AC=AD=1...if we give any other value to these sides then there will be problem to solve above said equations..

Vijay Sharma - 7 years, 1 month ago

Log in to reply

since ACD=ADC ISOSCELES TRIANGLE!

Sohan Samanta - 7 years, 1 month ago

Log in to reply

i know these two values are the same but how we say that these values are unit....

Vijay Sharma - 7 years, 1 month ago

Log in to reply

@Vijay Sharma Not a single scalar measure is given in the problem statement, so the number chosen for the length of AC and AD is irrelevant. The problem could probably have been solved with the assumption AC = AD = n, but that would make the simplifications and computations more complicated, so AC = AD = 1 is a nice choice.

John Mistele - 7 years, 1 month ago

good

Susanta Mondal - 7 years, 1 month ago

Nice approach !

engma dp - 7 years, 1 month ago

Brilliant!!!

Mariya Chorbova - 7 years, 1 month ago

Awesome Solution..Great Work!!!

Divya Dwivedi - 7 years, 1 month ago

I couldn´t remember the way to solve it. It was one of the most difficult parts of mathematics for me in high school :)

Rocio Pecero - 7 years, 1 month ago

let DC and EA intersects at P...so, angle aCd=aCp=48, angle cAe=cAp=63 than angle cPa=69... in the triangular EDP, angle ePd=69...coz angle ePd=cPa...and the triangular EDP is has ED=EP... therefore, angle cDe=pDe=ePd=69 degree.... in reply to @Eloy Machado ....ur assumption is useless and wrong...M can never ever be the midpoint of CD which is bisected by AE...and u cannot just LET AC=AD coz angle dAc is not equal to 90 degree...

Tanzim Khan - 7 years, 1 month ago

Log in to reply

Read CAREFULLY my solution, please.

Eloy Machado - 7 years, 1 month ago

Log in to reply

i read it...u r wrong...

Tanzim Khan - 7 years, 1 month ago

Log in to reply

@Tanzim Khan Read it again and again. Stress on CAREFULLY, please. The solution is correct. Verify with the solution given by One Top. Different approach same answer.

Mariya Chorbova - 7 years, 1 month ago

what technique you do sir?? i couldnt understand it..you're so GREAt

Renz Oliver Alfanta - 7 years, 1 month ago

84 < its wrong :(

ElectRon Shajal - 7 years, 1 month ago

This is wrong as CDE has many solution according to the angles BDE, BED and AED. I can give you number of combinations that can work. Try: BED, BDE, AED and CDE as 92, 70, 49 and 83 Try: BED, BDE, AED and CDE as 93, 69, 48 and 84 and so on If you want your solution will be: BED, BDE, AED and CDE as 90, 72, 51 and 81

Mohamed Elgammal - 7 years, 1 month ago

If u put the value CDE=54 degrees then the whole system matches.

Vikram Bhattacharjee - 7 years, 1 month ago

just get a protractor then measure it. if the correctness of the visual aid is 100% then the protractor can get an answer but if the visual aid is wrong then how can you give a correct given. hahahahaha.

Florimond Acop - 7 years, 1 month ago

48

Aqeel Ahmad - 7 years, 1 month ago

I solved it using simultaneous linear equations, with the data the small triangle in the mid has angles 69, cde and aed. Taking cde as x and aed as y, x+y=111, the next part is a bit unconventional. If you calculate the difference in angles for ecd and cda you get 18 which is the same result when you calculate the difference between angle ead and cea. Using this the difference between dca and x should be the same as that between cae and y. Therefore we get 48 - x=63 - y, which gets to y - x= 15

Solving these we get y=63, and x=48 and so the angle cde comes out as 48.

Aqeel Ahmad - 7 years, 1 month ago

Log in to reply

yaaa I too

Adarsh Sinha - 7 years ago

my solution is not by representing a mathematical solution ,.,i draw the illustrated triangle and i able to draw the exact measurement of all angles of the triangle by using an instrument that measures an angle which is a protractor ....i able to measure the angle of angle CDE which is 78 degree....

Victorio Elacion - 7 years, 1 month ago

My way is autocad

Nguyễn Văn Đức - 7 years, 1 month ago

very complicated ans

Manoj Mishra - 7 years, 1 month ago

I got 60 too without considering the proposed law, all conditions were satisfied.

Aleph Nesher - 7 years, 1 month ago

Some diagrams to support this answer would be great to follow...

Arun Jayapal - 7 years, 1 month ago

Yes, I have done this!

Khandaker Azad - 7 years, 1 month ago

nice solution :)

Naman Negi - 7 years, 1 month ago

panis lan q!

Jonnel Lauron - 7 years ago

my answer is 75 degrees and i used simple angle theorems and i got it :-) but 81, i think is wrong!!!!!!

Aayush Solanki - 7 years ago

sir how you could say that the angle of CDA = the angle of ACD? thx.

Rezki Noviana Agus - 7 years ago

angle DCE equal 30 as we subtracted angle DCA FROM ANGLE ACE 78-48 so we got 30 so angle DCE equal 30 since angle CED is a right angle its measur equal 90 so 180 -(30+90)= 60 so angle CDE EQUAL 60 degree .

Aia Elmaky - 7 years ago

ai vc fuma um baseado loco e resolve isso

Roger Marcus - 7 years ago

how can you assume it sir?

krishna havish - 7 years ago

@Prakash Mulabagal sir , i am a student of MPA , std. 9 . this problem is similar to the new year problem 2014 !!

Nihar Mahajan - 6 years, 5 months ago

Sir, how can you say AM is perpendicular to CD? if you trace all the angle. you get angle ACD=48 then angle AMC = 69

Taong Ma L - 7 years, 1 month ago

Log in to reply

Triangle ACD is isósceles, and my point M is the midpoint of CD, so AM is perpendicular to CD. Maybe your point M isn't the same as mine.

Eloy Machado - 7 years, 1 month ago

M is not the intersecting point of AE and CD. He assumes M is the midpoint of CD.

সালাম বাবু - 7 years, 1 month ago

it is wrong assumption

correct answer is 48 because angle ACD = angle EDC alternate angles

Chayan Bhawsar - 7 years, 1 month ago

Log in to reply

Alternate abgles happen only when a line intersects two paralell lines, here the sides of the triangle ABC are not parallel

Vamsi Adarsh - 7 years, 1 month ago

If AC =AR since triangleACD is isosceles,we should be able to draw acircle with A as midpoint and Radius = AC =AD.In which case BC is a tangent which is not possible as angle BCA not a right angle.Please correct me if I have erred.

Amit Chopra - 7 years, 1 month ago

Log in to reply

my mistake.BC is a secant and C becomes it's entry point to the circle.

Amit Chopra - 7 years, 1 month ago

Look at my steps, I think it makes sense and logically.

Your assumption: The midpoint is M.

From your answer we can get ∠AMC = 180° - ∠ACD - ∠EAC = 69°

Agree for this far?

Then, we can get ∠AMD = 180° - ∠AMC = 111°

Total of ΔADM = 180°

It makes sense, doesn’t it?

We know that ∠ECA + ∠EDA should be 180°

So, ∠CDE = 54°

Can u give me correction if I made a mistake, please?

Thank you

Yossie Yoshua - 7 years ago

Log in to reply

"From your answer we can get ∠AMC = 180° - ∠ACD - ∠EAC = 69° Agree for this far?" these lines are totally wrong.

Eka Kurniawan - 7 years ago

Log in to reply

can u explain? because that i know the sum of triangle AMC should be 180°

and then please look at my calculation

in triangle CAM, ∠CAM = 63°, ∠AMC = 69°, ∠MCA = 48°

in triangle ECM, ∠CEM = 39°, ∠EMC = 111°, ∠MCE = 30°

in triangle ADM, ∠AMD = 111°, ∠MDA = 48°, ∠DAM = 21°

in triangle EDM, ∠DEM = 57°, ∠EMD = 69°, ∠MDE = 54°

in triangle BED, ∠B = 18°, ∠BED = 84°, ∠BDE = 78°

Yossie Yoshua - 7 years ago

how do u come at AMC =180 - ACD - EAC.. look carefully AMC =180 - ACD - MAC

Ashwani Pandey - 7 years ago

Log in to reply

∠EAC = ∠MAC

and then 180 is total degree of triangle AMC

my calculation

in triangle CAM, ∠CAM = 63°, ∠AMC = 69°, ∠MCA = 48°

in triangle ECM, ∠CEM = 39°, ∠EMC = 111°, ∠MCE = 30°

in triangle ADM, ∠AMD = 111°, ∠MDA = 48°, ∠DAM = 21°

in triangle EDM, ∠DEM = 57°, ∠EMD = 69°, ∠MDE = 54°

in triangle BED, ∠B = 18°, ∠BED = 84°, ∠BDE = 78°

Yossie Yoshua - 7 years ago

Angle BAC= 60 degree. Angle acb= 60 dca +cad=cdb 90 degree cda=30 +60. =90. But we dont know about edc .So let edc be x. So because angle on straight line is 180. So thus, 90+x -90 +90 =180. which implies x=90. Angle cde=90.

HK Sid - 7 years ago

Log in to reply

The answer according to me maybe 90.

HK Sid - 7 years ago

easy to understand solution :)

Ashwani Pandey - 7 years ago
Ajit Athle
Apr 17, 2014

A Synthetic Solution along the lines of Langley's Adventitious Angles: Draw a line AG at 24° to AC meeting BC in G. Join G to D & observe that / AGC = 180 -78-24 =78°. So in tr. AGC we've AG = AC. Now / ADC = 180 -84 -48 = 48°. Thus, AD = AC = AG, But / DAG = 84 -24=60°. Thus, triangle ADG is equilateral. Now observe that / EAG =60-21=39° while / EGA=180 -78=102°; hence / GEA=180 -102 -39 =39°. This makes triangle EGA isosceles with GE = AG = GD. Now / EGD =102 -60 =42° which, in turn, means that / GDE=/ GED= (180-42)/2 = 69° and therefore / CDE = 180 -- 69 -30 = 81°. (Incidentally, /DEA = 30°, same as in Langley)

Yet another way to look at this: Once we locate G as above & prove that GA = GE = GD, we can say that a circle with G as centre and radius = GE will pass through points E, D & A. Now greater /_ AGE = 360 -102 =258° and thus / ADE=258/2 =129° which makes / CDE=129 -48 =81°.

Ajit Athle - 7 years, 1 month ago

Log in to reply

which angle is 102?

Haidar Islam Bhuiyan - 7 years, 1 month ago

Log in to reply

AG is drawn such that / CAG = 24° (meeting BC in G). Now / BCA=78° and / CAG=24°; hence / CGA = 180 -78-24=78° . So / BGA = / AGE =180 - 78 =102°

Ajit Athle - 7 years, 1 month ago

That is even better! Similar to Mercer's solution.

Mariya Chorbova - 7 years, 1 month ago

i think you guys, make it complicated. math is simple. my answer is 48. :)

Musdah Mulia - 7 years, 1 month ago

Very nice solution!

Eloy Machado - 7 years, 1 month ago

180-48-63=69 degrees

Faizan Khan - 7 years, 1 month ago

how u prove AD=AC=AG

Mohan Raj Chandra Sekar - 7 years, 1 month ago

how do u sketch a line at 24 and where the place is?

Haidar Islam Bhuiyan - 7 years, 1 month ago

Look at my steps, I think it makes sense and logically.

Your assumption: The midpoint is M.

From your answer we can get ∠AMC = 180° - ∠ACD - ∠EAC = 69°

Agree for this far?

Then, we can get ∠AMD = 180° - ∠AMC = 111°

Total of ΔADM = 180°

It makes sense, doesn’t it?

We know that ∠ECA + ∠EDA should be 180°

So, ∠CDE = 54°

Can u give me correction if I made a mistake, please?

Thank you

Yossie Yoshua - 7 years ago

A very neat and ingenious solution!

Joshua Lim - 7 years ago

Is there a solution not involving ACD being isoceles? All these Langley's problems are similar and rely on the same proof.

Maxence Seymat - 2 years, 4 months ago

G is not so arbitrary. Is the intersection between CE and the circle with center A and radius AC. Isn't it? Such a circle was already mentioned before after finding that AC=AD.

Ray Flores - 7 years, 1 month ago
Ahmad Saad
Oct 29, 2015

Ahmad, among all the solutions presented, I liked yours the most. However I was not able to follow the path and prove that Tr. CDF is equilateral. Could you please elaborate on that? Thanks, R. De Souza

Rogerio De Souza - 3 years, 2 months ago

Kya be chodu angle AEC= 39 HOGA chutiya

Shivendra Singh - 1 year, 11 months ago

extraordinary answer!

Andrew Lee - 1 year, 7 months ago
Xuming Liang
Sep 13, 2015

The problem can be generalized, here are the sufficient information I will use from this configuration: C A = D A , B C D = 3 0 , 2 C E A = A C B CA=DA, \angle BCD=30^{\circ}, 2\angle CEA=\angle ACB .

Construct F C F\ne C on B C BC such that A F = A C AF=AC , then F A E = A F E F E A = A C E C E A = C E A \angle FAE=\angle AFE-\angle FEA=\angle ACE-\angle CEA=\angle CEA , which means F A = F E FA=FE . In addition, observe that A A is the circumcenter of C F D \triangle CFD ; therefore F A D = 2 D C F = 6 0 \angle FAD=2\angle DCF=60^{\circ} and A F D \triangle AFD is equilateral. Hence F E = F A = F D FE=FA=FD , deducing that F F is the circumcenter of A D E \triangle ADE . Thus :

E D C = E D A C D A = 9 0 + C E A C D A \angle EDC=\angle EDA-\angle CDA=90^{\circ}+\angle CEA-\angle CDA

For this special configuration, E D C = 90 + 39 48 = 8 1 \angle EDC=90+39-48=\boxed {81^{\circ}}

As simple as it can be!

Satyajit Mohanty - 5 years, 9 months ago

Log in to reply

If it is simple enough can you please construct a diagram? Cause i am struggling

Asad Jawaid - 5 years ago

Log in to reply

Here you go!

https://i.imgur.com/METbYu2.png

Joshua Wijaya - 4 years, 8 months ago

Wrong, u cant generalize

Araldo Chierighini - 4 years ago

Using Sine Rule,

sin 3 9 sin 8 4 sin 6 3 sin 4 8 sin x = sin ( 3 0 + x ) \dfrac{\sin 39^\circ\sin 84^\circ}{\sin 63^\circ\sin 48^\circ}\sin x^\circ = \sin(30^\circ+x^\circ)

Solving, sin x = 0.987688 x = 8 1 \sin x = 0.987688\\\Large\boxed {x = 81^\circ}

How I got it? Here:

Let C D CD intersect E A EA at F F and E C = m C D = n C A = b C D E = x D E C = y EC = m\\CD = n\\CA = b\\\angle CDE = x\\\angle DEC = y Chasing angles, C E A = 3 9 E C D = 3 0 C A E = 6 3 C D A = 4 8 \angle CEA = 39^\circ\\\angle ECD = 30^\circ\\ \angle CAE = 63^\circ\\\angle CDA = ~48^\circ

Sine rule says, in a triangle, a sin B sin A = b \dfrac {a\sin B}{\sin A} = b

So, in C E A \triangle CEA , m sin 3 9 sin 6 3 = b \dfrac {m\sin 39^\circ}{\sin 63^\circ} = b

Now, in C D A \triangle CDA , b sin 8 4 sin 4 8 = n m sin 3 9 sin 6 3 sin 8 4 sin 4 8 = n \dfrac{b \sin 84^\circ}{\sin48^\circ} = n\\\\\Rightarrow \dfrac {m\sin 39^\circ}{\sin 63^\circ}\cdot\dfrac{\sin 84^\circ}{\sin48^\circ}=~n

In E D C \triangle EDC 18 0 ( x + 3 0 ) = y m sin y sin x = n 180^\circ - (x + 30^\circ) = y\\\dfrac{m\sin y}{\sin x} = ~n

sin ( 18 0 θ ) = sin θ \sin(180^\circ - \theta) = \sin\theta

So, we get m sin 3 9 sin 8 4 sin 6 3 sin 4 8 = m sin y sin x sin 3 9 sin 8 4 sin 6 3 sin 4 8 sin x = sin ( x + 3 0 ) m\dfrac {\sin 39^\circ\sin 84^\circ}{\sin 63^\circ\sin48^\circ} = m\dfrac{\sin y}{\sin x}\\\\\Rightarrow \dfrac{\sin 39^\circ\sin 84^\circ}{\sin 63^\circ\sin 48^\circ}\sin x = \sin(x+30^\circ)

Solve it to get the answer!

Moderator note:

Please use words to explain what you are doing. Not everyone can be a mind reader.

Please use words to explain what you are doing. Not everyone can be a mind reader.

Calvin Lin Staff - 5 years, 3 months ago

Log in to reply

I see. I was actually too lazy to do it... :)

Kishore S. Shenoy - 5 years, 2 months ago

Log in to reply

be good and tell us what did you do?

Aza Ama - 4 years, 9 months ago

Log in to reply

@Aza Ama There you go! I have given my solution!

Kishore S. Shenoy - 4 years, 9 months ago

Log in to reply

@Kishore S. Shenoy Thanks.

Instead of saying " sin x = 0.987 x = 8 1 \sin x = 0.987\ldots \Rightarrow x = 81^\circ ", you could
1. Explain why the answer is unique (in the relevant domain)
2. Show that x = 81 x = 81 satisfies the conditions.

Calvin Lin Staff - 4 years, 8 months ago

Log in to reply

@Calvin Lin I did not get you... can you explain?

Kishore S. Shenoy - 4 years, 8 months ago

In my solution I assumed the length of AC to be 1, then I used sine law and cosine law to find for the required angle.

Can you include the details? Thanks.

Calvin Lin Staff - 3 years, 2 months ago
Edwin Gray
Feb 28, 2018

From what is given, <CDA = 48, so triangle ACD is isosceles. Let CA = AD = 1. By the Law of sines, CD/ sin(84) = 1/ sin(48), so CD = 1.338261212.Filling some angles, < COA = 69, where O is the intersection of EA and CD., <EAD = 21, <BCD = 30, < CEA = 39. Define < AED by t. . Then by Sine Law, ED / sin(21, so ED, = .35836795/ sin(t). Also ED/sin(30) = CD/sin(39 + t) = 1/ sin(t). Using the value for CD and equating these two equations for ED , we can solve for t; t = 30, and from triangle EDO, we have x = 180 -t - 69 = 81. Ed Gray

Let AE and CD intersect at F. Following are the angles we get.
ACD.. =.. 48,......... ADC.. =.. 48,.......... ECD.. =.. 30,.......... EAC.. =.. 63,.......... EAD.. =... 21,..... AEC.. =.. 39 .......
EFC.. =.. DFA.. =.. 111, .......... DFE.. =.. AFC.. =.. 69,.......... CDE.. =.. X ........... DEA.. =.. Y.. =.. 111.. -.. X
Let AC = AD = a, since isosceles triangle CAD has two 48 angles.
Applying sin rule to the following triangles,
ACE :- EC/sin63 = a/sin39
CED :- EC/sinX = ED/sin30 ...............> sinX/EC = sin30/ED
AED :- a/sinY = ED/sin21...............................................................................................................................






( a/sinY ) ( sinX/EC ) ( EC/sin63 ) = ( a/sin39 ) ( sin30/ED) ( ED/sin21)
sinX/{ sin( 111 - X ) } = ( sin63/ sin39 ) * ( sin30/ sin21) .........................................................................

EXPANDING sin( 111 - X ) AND SIMPLIFYING WE GET X = 81.
Sorry I am not able to copy the sketch I have with me.

Look at my steps, I think it makes sense and logically.

Your assumption: The midpoint is M.

From your answer we can get ∠AMC = 180° - ∠ACD - ∠EAC = 69°

Agree for this far?

Then, we can get ∠AMD = 180° - ∠AMC = 111°

Total of ΔADM = 180°

It makes sense, doesn’t it?

We know that ∠ECA + ∠EDA should be 180°

So, ∠CDE = 54°

Can u give me correction if I made a mistake, please?

Thank you

Yossie Yoshua - 7 years ago

Log in to reply

may I know why ∠ECA + ∠EDA should be 180°? Isn't it that rule only apply when there is a quadrilateral in a circle?

reiko qi - 7 years ago

Log in to reply

the sum of opposite corner is 180°, isn't it? because ECAD is a quadrilateral, i thought the rule is valid for all quadrilaterals not only a quadrilateral in a circle like u told me,.. but when i try to count opposite corner of a trapezoid that has two angles 90°, yes the rule doesn't apply in all quadrilaterals. thanks.

but i am still sure my answer for ∠CDE = 54° makes sense when i count every angle in all triangles there, please give me more correction if u found out the mistakes. thanks again :)

my calculation

in triangle CAM, ∠CAM = 63°, ∠AMC = 69°, ∠MCA = 48°

in triangle ECM, ∠CEM = 39°, ∠EMC = 111°, ∠MCE = 30°

in triangle ADM, ∠AMD = 111°, ∠MDA = 48°, ∠DAM = 21°

in triangle EDM, ∠DEM = 57°, ∠EMD = 69°, ∠MDE = 54°

in triangle BED, ∠B = 18°, ∠BED = 84°, ∠BDE = 78°

Yossie Yoshua - 7 years ago

Log in to reply

@Yossie Yoshua No, the sum of the angles of opposite corners of a quadrilateral is 180° if and only if the quadrilateral is cyclic. Take a diamond for instance — as it becomes thinner, the sum of one pair of opposite angles increases to 360° while the sum of the other pair decreases to 0°, meaning that if the quadrilateral is not forced to be cyclic, then the sum of opposite angles is not constant. As for the answer you got, having all angles fit does not necessarily mean you have found the correct answer. This will become clear when you try to draw it out accurately with a protractor and straight lines —it is an impossible shape. In fact, substituting other random values for ∠CDE, and calculating the remaining angles from that value will also yield many plausible set of values for each angle like what you've found, but there is only one correct one (which happens to be ∠CDE = 81°). All the rest are impossible shapes with disconnected lines. Hope that helped :)

Joshua Lim - 7 years ago

81 is not the correct answer! because on substituting the value for the solution..and there after cross checking it... leads to a contradiction. sum angles of a triangle should be 180.. if we take 81 as angle CDE the sum of angles of a triangle BDE is just 138..

0 pending reports

×

Problem Loading...

Note Loading...

Set Loading...